Добавил:
Upload Опубликованный материал нарушает ваши авторские права? Сообщите нам.
Вуз: Предмет: Файл:
Алгебра_1 / Gilbert - Modern algebra with applications.pdf
Скачиваний:
34
Добавлен:
23.02.2015
Размер:
2.55 Mб
Скачать

Appendix 2

INTEGERS

The set Z = {0, ±1, ±2, ±3, . . .} of integers is essential to all of algebra, and has been studied for centuries. In this short section we derive the basic properties of Z, focusing on the idea of the greatest common divisor, and culminating in the prime factorization theorem. We assume a basic knowledge of the addition and multiplication of integers, and of their ordering.

INDUCTION

The following principle is an axiom for the set P = {1, 2, . . .} of positive numbers.

Well-Ordering Axiom. Every nonempty subset of P has a smallest member.

Our first deduction from the axiom gives a very useful method for proving sequences of statements are true.

Theorem 1. Induction Principle. Let p1, p2, . . . be statements such that:

(i)p1 is true.

(ii)If pk is true for some value of k 1, then pk+1 is true.

Then pn is true for every n 1.

Proof. Let X = {n 1|pn is false}. If X is nonempty, let m denote the smallest member of X (by the well-ordering axiom). Then m = 1 by (i), so if we write n = m 1, then n 1 and pn is true because n / X. But then pm = pn+1 is true

by (ii), a contradiction. So X is empty, as required.

 

Example 2. Prove Gauss’ formula: 1 + 2 + · · · + n = 21 n(n + 1) for n 1.

 

Modern Algebra with Applications, Second Edition, by William J. Gilbert and W. Keith Nicholson ISBN 0-471-41451-4 Copyright 2004 John Wiley & Sons, Inc.

INTEGERS

 

 

 

 

 

 

 

 

 

 

 

 

 

 

 

 

 

297

 

 

 

 

p

 

 

 

 

 

 

 

 

1

 

 

)

 

 

p

 

 

Proof. If

 

n denotes the statement 1 + 2 + · · · + n = 2 n(n + 1 1

then

 

1

is

true. If

p

k

holds, p

k+1

is true because 1

+

2

+ · · · +

k

+

(k

+

1)

= 2

k(k

+

1)

+

 

1

(k

 

 

 

 

 

 

 

 

 

(k + 1) =

2

+ 1)(k + 2). Hence every pk

is true by the induction principle.

 

 

 

 

 

 

 

 

 

 

 

 

 

 

 

 

 

 

 

 

 

 

There is nothing special about 1 in the induction principle. In fact, the list of statements can be started from any integer b.

Corollary 3. Extended Induction. If b is an integer, let pb , pb+1, . . . be statements such that:

(i)pb is true.

(ii)If pk is true for some value of k b, then pk+1 is true.

Then pn is true for every n b.

Proof. Apply the induction principle to show that the statements q1, q2, . . .

are all true, where qk is the statement that “pb+k1 is true.” This means that pb , pb+1, . . . are all true, as desired.

Sometimes it is convenient to be able to replace the inductive assumption that “pk is true” in Corollary 3 (ii) with the stronger assumption that “each of pb , pb+1, . . . , pk is true.” This is valid by the next theorem.

Theorem 4. Strong Induction. If b is an integer, let pb , pb+1, . . . be statements such that:

(i)pb is true.

(ii)If pb , pb+1, . . . , pk are all true for some value of k b then pk+1 is true.

Then pn is true for every n b.

Proof. Apply extended induction to the new statements qb , qb+1, . . ., where qk is the statement that “each of pb , pb+1, . . . , pk is true.”

An integer p is called a prime if p 2 and p cannot be written as a product of positive integers apart from p = 1 · p. Hence the first few primes are 2, 3, 5, 7, 11, 13, . . .. With a little experimentation, you can convince yourself that every integer n 2 is a product of (one or more) primes. Strong induction is needed to prove it.

Theorem 5. Every integer n 2 is a product of primes.

Proof. Let pn denote the statement of the theorem. Then p2 is clearly true.

If p2, p3, . . . , pk are all true, consider the integer k + 1.

If k + 1 is a prime,

there is nothing to prove. Otherwise, k + 1 = ab, where 2

a, b k. But then

each of a and b are products of primes because pa and pb are both true by the (strong) induction assumption. Hence ab = k + 1 is also a product of primes, as required.

298

INTEGERS

Corollary 6. Euclid’s Theorem. There are infinitely many primes.

Proof. Suppose on the contrary that p1, p2, . . . , pm are all the primes. In that case, consider the integer n = 1 + p1p2 · · · pm. It is a product of primes by Theorem 5, so is a multiple of pi for some i = 1, 2, . . . , m. But then 1 is an integral multiple of pi , a contradiction.

Another famous question concerns twin primes, that is, consecutive odd numbers that are both primes: 3 and 5, 5 and 7, 11 and 13, . . .. The question is whether there are infinitely many twin primes. One curious fact which suggests

 

1

 

 

 

 

 

 

 

 

1

 

 

that there may be only finitely many is that the series

 

p

p a twin prime of

 

 

 

 

p

 

 

 

 

 

 

 

 

 

reciprocals of twin primes is convergent, whereas the series

 

 

p a prime

 

 

 

 

 

 

 

 

of all prime reciprocals is known to be divergent. But the question remains

open.

Euclid’s theorem certainly implies that there are infinitely many odd primes, that is, primes of the form 2k + 1, where k 1. A natural question is whether if a and b are positive integers, there are infinitely many primes of the form ak + b, k 1. This clearly cannot happen if a and b are both multiples of some integer greater than 1. But it is true if 1 is the only positive common divisor of a and b, a famous theorem first proved by P. G. L. Dirichlet (1805–1859).

DIVISORS

When we write fractions like 22 17 we are using the fact that 22 = 3 · 7 + 1; that is, when 22 is divided by 7 there is a remainder of 1. The general form of this observation is fundamental to the study of Z.

Theorem 7. Division Algorithm . Let n and d 1 be integers. There exist uniquely determined integers q and r such that

n = qd + r and 0 r < d.

Proof. Let X = {n t d|t Z, n t d 0}. Then X is nonempty (if n 0, then n X; if n < 0, then n(1 d) X). Hence let r be the smallest member of X (by the well-ordering axiom). Then r = n qd for some q Z, and it remains to show that r < d. But if r d, then 0 r d = n (q + 1)d, so r d is in X contrary to the minimality of r.

As to uniqueness, suppose that n = q d + r , where 0 r < d. We may assume that r r (a similar argument works if r r). Then 0 r r = (q q)d, so (q q)d is a nonnegative multiple of d that is less than d (because

r r r < d). The only possibility is (q q)d = 0,

so q = q, and hence

r = r.

 

This as not an algorithm at all, it is a theorem, but the name is well established.

INTEGERS

299

Given n and d 1, the integers q and r in Theorem 7 are called, respectively, the quotient and remainder when n is divided by d. For example, if we divide n = −29 by d = 7, we find that 29 = (5) · 7 + 6, so the quotient is 5 and remainder is 6.

The usual process of long division is a procedure for finding the quotient and remainder for a given n and d 1. However, they can easily be found with a

calculator. For example, if n = 3196 and d = 271 then n = 11.79 approximately, d

so q = 11. Then r = n qd = 215, so 3196 = 11 · 271 + 215, as desired.

If d and n are integers, we say that d divides n, or that d is a divisor of n, if n = qd for some integer q. We write d|n when this is the case. Thus, a positive integer p is prime if and only if p has no positive divisors except 1 and p. The following properties of the divisibility relation | are easily verified:

(i)n|n for every n.

(ii)If d|m and m|n, then d|n.

(iii)If d|n and n|d, then d = ±n.

(iv)If d|n and d|m, then d|(xm + yn) for all integers x and y.

These facts will be used frequently below (usually without comment).

Given positive integers m and n, an integer d is called a common divisor of m and n if d|m and d|n. The set of common divisors of m and n clearly has a maximum element; what is surprising is that this largest common divisor is actually a multiple of every common divisor. With this in mind, we make the following definition: If m and n are integers, not both zero, we say that d is the greatest common divisor of m and n, and write d = gcd(m, n), if the following three conditions are satisfied:

(i)d 1.

(ii)d|m and d|n.

(iii)If k|m and k|n, then k|d.

In other words, d = gcd(m, n) is a positive common divisor that is a multiple of every common divisor. It is routine to use conditions (i) to (iii) to show that d is unique if it exists. Note that d does not exist if m = 0 = n, but it does exist in every other case (although this is not apparent). In fact, even more is true:

Theorem 8. Let m and n be integers, not both zero. Then d = gcd(m, n) exists, and d = xm + yn for some integers x and y.

Proof. Let X = {sm + t n|s, t Z; sm + t n 1}. Then X is not empty since m2 + n2 is in X, so let d be the smallest member of X (by the well-ordering axiom). Since d X we have d 1 and d = xm + ym for integers x and y, proving conditions (i) and (iii) in the definition of the gcd. Hence it remains to show that d|m and d|n. We show that d|n; the other is similar. By the division algorithm

300 INTEGERS

write n = qd + r, where 0 r < d. Then r = n q(xm + yn) = (qx)m +

(1 qy)n. Hence, if r 1, then r X, contrary to the minimality

of d. So

r = 0 and we have d|n.

 

When gcd(m, n) = xm + yn where x and y are integers, we say that gcd(m, n) is a linear combination of m and n. There is an efficient way of computing x and y using the division algorithm. The following example illustrates the method.

Example 9. Find gcd(37, 8) and express it as a linear combination of 37 and 8.

Proof. It is clear that gcd(37, 8) = 1 because 37 is a prime; however, no linear combination is apparent. Dividing 37 by 8, and then dividing each successive divisor by the preceding remainder, gives the first set of equations. The last

37 = 4 · 8 + 5 1 = 3 1 · 2 = 3 1(5 1 · 3)

8 = 1 · 5 + 3

= 2 · 3 5 = 2(8 1 · 5) 5

5 = 1 · 3 + 2

= 2 · 8 3 · 5 = 2 · 8 3(37 4 · 8)

3 = 1 · 2 + 1

= 14 · 8 3 · 37

2 = 2 · 1

 

nonzero remainder is 1, the greatest common divisor, and this turns out always to be the case. Eliminating remainders from the bottom up (as in the second set of equations) gives 1 = 14 · 8 3 · 37.

The method in Example 9 works in general.

Theorem 10. Euclidean Algorithm. Given integers m and n 1, use the division algorithm repeatedly:

m = q1n + r1

0 r1 < n

n = q2r1 + r2

0 r2 < r1

r1 = q3r2 + r3

0 r3 < r2

.

.

 

.

.

 

.

.

 

rk2 = qk rk1 + rk

0 rk < rk1

rk1 = qk+1rk

 

where in each equation the divisor at the preceding stage is divided by the remainder. These remainders decrease

r1 > r2 > · · · 0